Một số bài toán

Một phần của tài liệu một số vấn đề về căn nguyên thủy và ứng dụng (Trang 47 - 66)

3 MỘT SỐ ỨNG DỤNG CỦA CĂN NGUYÊN THỦY

3.4 Một số bài toán

Mục này trình bày lời giải một số bài toán số học xuất hiện trong các đề thi học sinh giỏi bằng cách sử dụng căn nguyên thủy. Các kết quả trong mục này được tham khảo từ các tài liệu [1], [2], [4], [6].

Bài 1. (Romania TST 1996) Tìm các số nguyên tố p, q thỏa mãn

α3pq ≡α (mod 3pq) với mọi số nguyên α.

Lời giải. Xét số nguyên α với (3pq, α) = 1, ta có α3pq−1 ≡1 (mod 3pq). Nếu α= 2 là một căn nguyên thủy theo môđun 3 thì

từ đó suy ra 2|3pq−1. Do đó p, q là số lẻ.

Nếu α là một căn nguyên thủy theo môđun p (do p là một số nguyên tố) thì

αp−1≡1 (mod p) và α3pq−1≡1 (mod p) từ đó suy ra p−1|3pq−1.

Nếu α là một căn nguyên thủy theo môđun q (do q là số nguyên tố) thì

αq−1 ≡1 (mod q) và α3pq−1≡1 (mod q) từ đó suy raq−1|3pq−1. Khi đó, vì 3qp−1 q−1 = 3p+ 3p−1 q−1 cho nên 3p−1 q−1 cũng là số nguyên. Không mất tính tổng quát, giả sử p≤q.

Nếu p = q thì 3p−1

q−1 = 3 + 2

q−1. Vậy p = q = 3. Thử lại, với α = 2, ta có 29≡86≡2(mod9), điều này mâu thuẫn với giả thiết. Như vậyp < q. Vìp≤q−2 cho nên 3p−1 q−1 ≤ 3q−7 q−1 < 3q−3 q−1 = 3. Vậy 3p−1 q−1 = 1 hoặc 3p−1 q−1 = 2. Tuy nhiên rõ ràng 3p−1 q−1 6= 1. Thật vậy vì 3p−1 =q−1nên q = 3p, trái với giả thiết q là số nguyên tố. Khi đó, 3p−1

q−1 = 2 hay q= 3p+ 1 2 . Mặt khác 3qp−1 p−1 = 3q+ 3q−1 p−1 là số nguyên từ đó suy ra 3q−1 p−1 cũng là số nguyên. Do đó 9p+ 1 2(p−1) = 9 2 + 10 2(p−1)

là số nguyên. Vì p−1|10 cho nên p= 11, q= 17 hoặc p= 3, q= 5.

Thử lại, với p= 11, q = 17, ta có 3qp= 561. Xét số nguyên α với(561, α) = 561. Khi đó 561 |α từ đó suy ra α561≡0≡α (mod 561), thỏa mãn yêu cầu đề bài.

Xét số nguyên α với 1<(561, α) =d <561, ta có sáu trường hợp sau.

(mod 17) từ đó suy ra α80 ≡ 1 (mod 11.17) hay α560 ≡ 1 (mod 11.17). Do đó

α561 ≡α (mod 11.17) và α561 ≡0≡α (mod 3). Vậy α561≡α (mod 561).

Trường hợp 2: Nếu d = 11 thì (3.17, α) = 1. Khi đó α2 ≡ 1 (mod 3) và α16 ≡ 1 (mod 17)từ đó suy raα16≡1 (mod 3.17)hayα560 ≡1 (mod 3.17). Do đóα561 ≡α

(mod 3.17) và α561≡0≡α (mod 11). Vậy α561≡α (mod 561).

Trường hợp 3: Nếu d = 17 thì (3.11, α) = 1. Khi đó α2 ≡ 1 (mod 3) và α10 ≡ 1 (mod 11)từ đó suy raα10≡1 (mod 3.11)hayα560 ≡1 (mod 3.11). Do đóα561 ≡α

(mod 3.11) và α561≡0≡α (mod 17). Vậy α561≡α (mod 561).

Trường hợp 4: Nếu d = 3.11 thì (17, α) = 1. Khi đó α16 ≡ 1 (mod 17) từ đó suy ra α560 ≡ 1 (mod 17). Do đó α561 ≡ α (mod 17) và α561 ≡0 ≡α (mod 3.11). Vậy

α561 ≡α (mod 561).

Trường hợp 5: Nếu d = 3.17 thì (11, α) = 1. Khi đó α10 ≡ 1 (mod 11) từ đó suy ra α560 ≡ 1 (mod 11). Do đó α561 ≡ α (mod 11) và α561 ≡0 ≡α (mod 3.17). Vậy

α561 ≡α (mod 561).

Trường hợp 6: Nếu d = 11.17 thì (3, α) = 1. Khi đó α2 ≡ 1 (mod 3) từ đó suy ra α560 ≡ 1 (mod 3). Do đó α561 ≡ α (mod 3) và α561 ≡ 0 ≡ α (mod 11.17). Vậy

α561 ≡α (mod 561). Vậy cặp số p, q thỏa mãn yêu cầu đề bài.

Xét số nguyên α với (561, α) = 1. Khi đó α560 = (α2)280 ≡ 1 (mod 3),

α560 = (α10)56≡1 (mod 11)và α560= (α16)35≡1 (mod 17). Từ đó suy raα560≡1 (mod 561) hay α561≡α (mod 561), thỏa mãn yêu cầu đề bài.

Nếu p = 3, q = 5 thì 3qp = 45. Xét α = 2, khi đó 245 ≡ 17 (mod 45), điều này mâu thuẫn với giả thiết. Như vậy, bài toán có một đáp án là p= 11, q= 17.

Bình luận. Rõ ràng việc áp dụng căn nguyên thủy vào bài toán này giúp ta tìm được các đặc trưng của p và q một cách dễ dàng và nhanh chóng.

Bài 2. (Putnam 1994) Cho số nguyên không âm a bất kỳ, đặt

na = 101a−100.2a.

Chứng minh rằng với 0≤a, b, c, d≤99 và na+nb≡nc+nd (mod 10100) thì

{a, b}={c, d}.

Lời giải. Theo giả thiết, ta có

na+nb ≡nc+nd (mod 10100) từ đó suy ra

101a−100.2a+ 101b−100.2b ≡101c−100.2c+ 101d−100.2d (mod 10100) do đó

101(a+b−c−d)≡100(2a+ 2b−2c−ad) (mod 101.100)

Khi đó a+b ≡c+d (mod 100) và 2a+ 2b ≡2c+ 2d (mod 101). Mặt khác, vì 2 là một căn nguyên thủy theo môđun 101 cho nên 2a+b≡2c+d (mod 101). Do đó (2c−2a)(2c−2b) = 22c−(2a+ 2b)2c+ 2a+b≡22c−(2c+ 2d)2c+ 2c+d = 0 (mod 101). Vì 101 là một số nguyên tố cho nên 2c ≡2a (mod 101) hoặc 2c ≡ 2b (mod 101). Do a và b có vai trò như nhau từ đó suy ra không mất tính tổng quát, giả sử 2c ≡ 2a (mod 101). Khi đó 2b ≡ 2d (mod 101). Vì 2 là một căn nguyên thủy theo môđun 101 cho nên a ≡c (mod 100) và b ≡ d (mod 100) hay {a, b}={c, d}

do 0 ≤ a, b, c, d ≤ 99. Chứng minh tương tự với 2c ≡ 2b (mod 101), ta cũng có

{a, b}={c, d}. Vậy ta có điều phải chứng minh.

Bình luận. Từ giả thiết, ta thấy được mấu chốt của bài toán nằm ở 10100 = 101.100 và ϕ(101) = 100. Đặc biệt, số 2 là một căn nguyên thủy theo môđun 101. Từ các dữ kiện này, dễ thấy việc áp dụng căn nguyên thủy là phương pháp nên được chọn để giải quyết bài toán.

Bài 3. (USEMO lần thứ nhất) Chứng minh rằng với mọi số nguyên tố p, tồn tại số nguyên dương n sao cho:

1n+ 2n−1+. . .+n1 ≡2020 (modp).

Lời giải. Theo Định lý Euler, với mọi số nguyên tố p và số nguyên a, (a, p) = 1 ta có ap−1 ≡1 (mod p) từ đó suy ra ap(p−1)−a+1≡ap−a (mod p). Khi đó

p P a=1 ap(p−1)−a+1 ≡ p−1 P a=1 ap−a (mod p) p P a=1 (p+a)p(p−1)−p−a+1≡ p−1 P a=1 ap−a−1 (mod p) . . . p P a=1 ((p−2)p+a)p(p−1)−(p−2)p−a+1≡ p−1 P a=1 ap−a+1 (mod p). Cộng vế theo vế p(p−1) P a=1 ap(p−1)−a+1≡ p−1 P a=1 p−1 P i=1 ai (mod p).

Vìp là một nguyên tố cho nên tồn tại số nguyêng là một căn nguyên thủy theo môđunpvà {1, g, g2, . . . , gp−2}là hệ thặng dư thu gọn theo môđun p. Khi đó, với mọi số nguyên a, 1 ≤ a ≤ p−1, tồn tại số nguyên j với 1 ≤ j ≤ p−1 sao cho

a≡gj (mod p). Với a= 1 p−1 P i=1 ai ≡p−1≡ −1 (mod p). Với a6= 1 p−1 P i=1 ai≡ p−2 P i=0 gij = g (p−1)j−1 gj−1 ≡0 (mod p). Do đó, p(p−1) P a=1 ap(p−1)−a+1 ≡ −1 (mod p) hay kp(p−1) P a=1 akp(p−1)−a+1≡k p(p−1) P a=1 ap(p−1)−a+1≡ −k (mod p).

Vậy ta chọn n = −2020p(p−1) khi đó 1n + 2n−1+. . .+n1 ≡ 2020 (modp) theo yêu cầu của đề bài.

Bình luận. Vấn đề khó khăn ở bài toán là số liệu rất lớn và phức tạp. Việc áp dụng căn nguyên thủy, cụ thể là ứng dụng chỉ số, vào bài toán giúp số liệu trở nên đơn giản hơn rất nhiều so với ban đầu.

Bài 4. (Hungary - Israel Math Competition 2009) Cho số nguyên tố p. Tìm tất cả các số nguyên dương k sao cho

Sk = 1k + 2k+. . .+ (p−1)k chia hết cho p.

Lời giải. Vì p là một số nguyên tố cho nên tồn tại g là một căn nguyên thủy theo môđun p. Khi đó gp−1 ≡1 (mod p)và {1, g, g2, . . . , gp−2} là một hệ thặng dư thu gọn theo môđun p từ đó suy ra Sk ≡1 +gk +. . .+g(p−2)k (mod p).

Nếu p−1 | k thì gk ≡ 1 (mod p). Do đó Sk ≡ 1 + 1 +. . .+ 1 = p−1 (mod p), điều này mâu thuẫn với giả thiết.

Nếu p−1- k thì gk 6≡1 (mod p) và g(p−1)k ≡1 (mod p). Khi đó

Sk ≡ g

(p−1)k−1

gk−1 ≡0 (mod p), thỏa mãn yêu cầu đề bài.

Như vậy, k là các số nguyên dương không chia hết cho p−1.

Bình luận. Thực chất nhờ bài toán 4 mà chúng tôi giải quyết được bài toán số 3. Rõ ràng việc áp dụng căn nguyên thủy, cụ thể ở đây là ứng dụng chỉ số, vào bài toán giúp số liệu ban đầu từ phức tạp trở nên đơn giản hơn nhiều.

Bài 5. (Iran TST 2013) Cho p là một số nguyên tố lẻ. Chứng minh rằng tồn tại một số tự nhiên x sao cho x và 4x đều là căn nguyên thủy theo môđun p.

Lời giải. Vì p nguyên tố cho nên tồn tại số nguyên dương g là căn nguyên thủy theo môđunp, (g, p) = 1và {1, g, g2, . . . , gp−2}là hệ thặng dư thu gọn theo môđun

p. Khi đó tồn tại số nguyên dương r, 0≤r ≤p−2 với gr ≡2 (mod p) từ đó suy rag2r ≡4 (mod p). Gọi p1, p2, . . . , pl là các ước nguyên tố phân biệt củap−1. Với mỗi số k nguyên dương sao cho 1≤k≤l, đặt mk là số nguyên dương sao cho

mk 6≡0 (mod pk) và mk 6≡ −2r (mod pk).

Theo Định lý phần dư Trung hoa, tồn tại một số tự nhiên m sao cho

m≡mk (mod pk), ∀1≤k ≤l.

Vì m và m+ 2r đều không chia hết cho pk cho nên m và m+ 2r đều nguyên tố cùng nhau với p−1. Từ đó suy ra các số m,2m,3m, . . . ,(p−2)m đều không chia hết cho p−1, các số (m+ 2r),2(m+ 2r),3(m+ 2r), . . . ,(p−2)(m+ 2r) cũng không chia hết cho p−1. Mặt khác, vì g là một căn nguyên thủy theo môđun p cho nên ta được

gm, g2m, . . . , g(p−2)m 6≡1 (mod p)

gm+2r, g2(m+2r), . . . , g(p−2)(m+2r) 6≡1 (mod p).

Do đógm và gm+2r là các căn nguyên thủy theo môđunp. Vậy chọn sốx=gm và 4x≡4gm ≡gm+2r (mod p). Khi đó, x và 4x là các căn nguyên thủy theo môđun

p cần tìm.

Bình luận. Bài toán nêu trực tiếp về vấn đề căn nguyên thủy, do đó cần dùng các tính chất về căn nguyên thủy của x và ứng dụng chỉ số chứng minh tồn tại 4x cũng thỏa mãn điều kiện là một căn nguyên thủy.

Bài 6. (IMO Shortlist 1997)Chứng minh rằng nếu trong một cấp số cộng vô hạn gồm các số nguyên dương chứa một số chính phương và một số lập phương thì dãy số này cũng chứa một lũy thừa sáu của một số nguyên.

Lời giải. Gọi cấp số cộng gồm các số nguyên dương là a+qn với a, q và n là các số nguyên, trong đó 0≤a≤q−1, q và n là các số dương. Theo giả thiết, tồn tại

các số nguyên dương x và y sao cho x2=a+qnx và y3=a+qny với nx và ny là các số nguyên dương. Khi đó

x2 ≡a (mod q) và y3 ≡a (mod q).

Xét a = 0, chọn x2 = q2 với nx = q, y3 =q3 với ny =q2 và dãy số có z6 = q6 với

nz = q5, ta có điều phải chứng minh. Xét 1 ≤ a ≤ q−1. Với q = 1 ta có dãy số nguyên vô hạn, trường hợp này hiển nhiên đúng. Với q = pα với p là một số nguyên tố, α là một số nguyên dương, ta có hai trường hợp sau.

Trường hợp 1: Xét q = pα lẻ từ đó suy ra p là một số nguyên tố lẻ. Khi đó tồn tại số nguyên g là một căn nguyên thủy theo môđun q. Nếu (a, q) > 1 thì (a, q) = pk với k là một số nguyên dương, 1≤k ≤α. Khi đó

x2=a+qnx=pk(a0+pα−knx)

với (a0, p) = 1 từ đó suy ra p - (a0 +pα−knx), do đó pk là số chính phương hay 2|k. Từ đó suy ra x2≡a (mod q) hay

x pk2

2

≡a0 (mod q0 =pα−k). Chứng minh tương tự như trên, ta cũng có 3|k và

y pk3

3

≡a0 (mod q0 =pα−k).

Như vậy, ta rút gọn được phương trình đồng dư cho pk để a0 và q0 nguyên tố cùng nhau mà không làm ảnh hưởng đến giả thiết ban đầu của bài toán. Do đó, không mất tính tổng quát, ta xét (a, q) = 1. Vì a vừa là một thặng dư bậc hai vừa là một thặng dư bậc ba theo môđun q cho nên tồn tại các số nguyên t và s

sao cho

Khi đó 2t≡3s (mod ϕ(q) = (p−1)pα−1). Vì p−1 chẵn cho nên 2|s hay tồn tại số nguyênz sao cho s= 2z. Do đóa ≡g3(2z) = (gz)6 (mod q). Như vậy ta có điều phải chứng minh.

Trường hợp 2: Xét q = 2α. Với q = 2, cấp số cộng là a+ 2n. Nếu a chẵn thì cấp số cộng là dãy số tự nhiên chẵn, trường hợp này hiển nhiên đúng. Ngược lại nếu a lẻ thì cấp số cộng là dãy số tự nhiên lẻ, trường hợp này cũng hiển nhiên đúng.

Với q = 4. Nếu a ≡1 (mod 4) thì ta có 32,53,76 nằm trong cấp số cộng. Nếu

a≡2 (mod 4) thì tồn tại số nguyên h với a= 4h+ 2 từ đó suy ra

x2 = 4h+ 2 + 4nx = 2(2(h+nx) + 1),

điều này dẫn đến mâu thuẫn. Như vậy trường hợp này dãy không chứa số chính phương. Nếu a ≡3 (mod 4) thì điều này dẫn đến mâu thuẫn. Thật vậy, vì 3 là một căn nguyên thủy theo môđun 4cho nên 3 là một bất thặng dư bậc hai theo môđun 4 hay a cũng là một bất thặng dư bậc hai theo môđun 4. Do đó trường hợp này dãy không chứa số chính phương.

Xét q = 2α với α là số nguyên dương lớn hơn 2. Tương tự như trường hợp

q=pα, không mất tính tổng quát, giả sử(a, q) = 1hay alẻ. Theo Mệnh đề 1.2.5, các số (−1)u5v với 0≤u≤1 và 0≤v ≤2α−2 lập thành một hệ thặng dư thu gọn theo môđun 2α. Vì x2≡a (mod 2α) cho nên x lẻ. Khi đó tồn tại số nguyên v sao cho x≡ ±5v (mod 2α) từ đó suy ra

a≡x2≡52v (mod 2α).

Tương tự, ta cũng có y lẻ từ đó suy ra tồn tại số nguyên t với 0≤ t≤ 2α−2 sao cho y≡ ±5t (mod 2α), do đó

a≡y3≡ ±53t (mod 2α). Khi đó 54v ≡a2 ≡56t (mod 2α) từ đó suy ra

4v ≡6t (mod 2α−2) hay 2v ≡3t (mod 2α−3).

Nếu α >3 thì 2|t hay tồn tại số nguyên m sao cho t = 2m. Khi đó

a≡56m= (5m)6 (mod 2α),

ta có điều phải chứng minh. Nếu α = 3 thì q = 8. Khi đó hệ thặng dư thu gọn theo môđun 8 là −52,−5,5,52. Vì vậy với mọi x lẻ bất kỳ, ta có x2 ≡1 (mod 8) từ đó suy ra a = 1. Khi đó, chọn y ≡ 52 (mod 8) do y3 ≡ 1 = a (mod 8). Vậy

a≡y3 ≡56 (mod 8), ta có điều phải chứng minh. Với q = pα1

1 pα2

2 · · ·pαk

k với pi là các ước số nguyên tố phân biệt, αi là các số nguyên dương, 1≤i < k. Khi đó

x2 ≡a (mod q) và y3 ≡a (mod q). Từ đó suy ra

x2 ≡a (mod pαi

i ) và y3≡a (mod pαi

i ) với mọi 1≤i≤k.

Khi đó, từ trường hợp q=pα, ta có kết quả luôn tồn tại số nguyên zi sao cho

a≡zi6 (mod pαi i ).

Theo định lý thặng dư Trung Hoa, luôn tồn tại số nguyên dương n sao cho

n ≡zi (mod pαi

i ) với mọi 1≤i≤k. Do đó

n6≡zi6 ≡a (mod pαi i )

từ đó suy ra n6≡a (mod q). Vậy ta có điều phải chứng minh.

Bình luận. Vì bài toán này liên quan trực tiếp đến thặng dư bậc hai và thặng dư bậc ba nên phương pháp áp dụng căn nguyên thủy, cụ thể là ứng dụng chỉ

số, là phù hợp. Tuy nhiên cần chú ý phân thành nhiều trường hợp vì không phải số nguyên nào cũng tồn tại căn nguyên thủy.

Bài 7. (Trung Quốc TST 1993) Với mỗi số nguyên tố p lẻ, ta định nghĩa

F(p) = p−1 2 P k=1 k120 và f(p) = 1 2 − F(p) p . Tìm giá trị của f(p).

Lời giải. Vì p là một số nguyên tố cho nên tồn tại số nguyên dương g là một căn nguyên thủy theo môđun p. Khi đó gp−1 ≡ 1 (mod p) và {1, g, g2, . . . , gp−2}

là một hệ thặng dư thu gọn theo môđun p. Do đó, với mọi số nguyên dương k, 1≤k≤p−1, tồn tại số nguyên dương i, 1≤i≤p−1 sao cho k≡gi (mod p).

Nếu p−1- 120 thì g1206≡1 (mod p) và g120(p−1) ≡1 (mod p). Như vậy

F(p) = p−1 2 X k=1 k120= 1 2 p−1 2 X k=1 k120+k120≡ 1 2 p−1 2 X k=1 k120+ (p−k)120 ≡ 1 2 p−1 X i=1 g120i = g 120(g120(p−1) −1) 2(g120−1) ≡0 (mod p). Từ đó suy ra f(p) = 1 2.

Nếu p−1|120 thì k120 ≡1 (mod p) với mọi 1≤k ≤ p−1

2 . Do đó F(p)≡ p−1 2 (mod p) từ đó suy ra f(p) = 1 2− p−1 2p = 1 2p.

Bình luận. Tương tự bài 4, bài 7 cũng là một ví dụ điển hình cho việc áp dụng căn nguyên thủy hay cụ thể là chỉ số làm cho bài toán trở nên đơn giản hơn.

Bài 8. (IOM Shortlist 2012) Chứng minh rằng với mọi số nguyên tố p > 100 và mọi số nguyên r luôn tồn tại hai số nguyên dương a, b sao cho p|a2+b5−r.

nguyên thủy theo môđun p. Nếu 5 - p−1 thì g5 cũng là một căn nguyên thủy theo môđunp và {1, g5,(g5)2, . . . ,(g5)p−2} là hệ thặng dư thu gọn theo môđun p. Khi đó, tồn tại số i với 0≤i≤p−2 sao cho

r≡(g5)i (mod p).

Ta chọn b =gi (mod p) và a = 0, ta có a2+b5 ≡ g5i ≡ r (mod p). Vậy ta có điều phải chứng minh.

Vì p là một số nguyên tố lớn hơn 100 nên p lẻ hay 2| p−1. Nếu 5 |p−1 thì

p= 10k+ 1 với k là một số nguyên k ≥10. Xét p| r, chọn a= b = 0, ta có điều

Một phần của tài liệu một số vấn đề về căn nguyên thủy và ứng dụng (Trang 47 - 66)

Tải bản đầy đủ (PDF)

(66 trang)